Los Angeles Pierce College Role of Political Parties in Society Questions

Chapter 9 – Political Parties

1) America runs on the two-party system. Especially now, we see the intense rivalry between the Democrats and Republicans. Explain the role political parties play in elections.

2) Discuss how political parties were first formed. Use examples from the text.

Chapter 10 – Interest groups and lobbying

3) Provide some details about what interest groups are; and also how they are affected by one’s socio-economic status.

Chapter 11 – Congress

4) Now that we are heading into in-depth study of the 3 main branches of our government, discuss the formation and role of Congress.

Chapter 9 Political Parties Figure 9.1 The families of the 2012 presidential candidates joined in the
festivities at the Democratic National Convention in Charlotte, North Carolina, (left) and the Republican
National Convention in Tampa, Florida (right). (credit right: modification of work by “PBS
NewsHour”/Flickr) Chapter Outline 9.1 What Are Parties and How Did They Form? 9.2 The Two-Party
System 9.3 The Shape of Modern Political Parties 9.4 Divided Government and Partisan Polarization
Introduction In 2012, Barack Obama accepted his second nomination to lead the Democratic Party into
the presidential election (Figure 9.1). During his first term, he had been attacked by pundits for his
failure to convince congressional Republicans to work with him. Despite that, he was wildly popular in
his own party, and voters reelected him by a comfortable margin. His second term seemed to go no
better, however, with disagreements between the parties resulting in government shutdowns and the
threat of credit defaults. Yet just a few decades ago, then-president Dwight D. Eisenhower was criticized
for failing to create a clear vision for his Republican Party, and Congress was lampooned for what was
deemed a lack of real conflict over important issues. Political parties, it seems, can never get it right—
they are either too polarizing or too noncommittal. While people love to criticize political parties, the
reality is that the modern political system could not exist without them. This chapter will explore why
the party system may be the most important component of any true democracy. What are political
parties? Why do they form, and why has the United States typically had only two? Why have political
parties become so highly structured? Finally, why does it seem that parties today are more polarized
than they have been in the past? Chapter 9 | Political Parties 327 9.1 What Are Parties and How Did
They Form? Learning Objectives By the end of this section, you will be able to: • Describe political
parties and what they do • Differentiate political parties from interest groups • Explain how U.S. political
parties formed At some point, most of us have found ourselves part of a group trying to solve a
problem, like picking a restaurant or movie to attend, or completing a big project at school or work.
Members of the group probably had various opinions about what should be done. Some may have even
refused to help make the decision or to follow it once it had been made. Still others may have been
willing to follow along but were less interested in contributing to a workable solution. Because of this
disagreement, at some point, someone in the group had to find a way to make a decision, negotiate a
compromise, and ultimately do the work needed for the group to accomplish its goals. This kind of
collective action problem is very common in societies, as groups and entire societies try to solve
problems or distribute scarce resources. In modern U.S. politics, such problems are usually solved by
two important types of organizations: interest groups and political parties. There are many interest
groups, all with opinions about what should be done and a desire to influence policy. Because they are
usually not officially affiliated with any political party, they generally have no trouble working with
either of the major parties. But at some point, a society must find a way of taking all these opinions and
turning them into solutions to real problems. That is where political parties come in. Essentially, political
parties are groups of people with similar interests who work together to create and implement policies.
They do this by gaining control over the government by winning elections. Party platforms guide
members of Congress in drafting legislation. Parties guide proposed laws through Congress and inform
party members how they should vote on important issues. Political parties also nominate candidates to
run for state government, Congress, and the presidency. Finally, they coordinate political campaigns and
mobilize voters. POLITICAL PARTIES AS UNIQUE ORGANIZATIONS In Federalist No. 10, written in the late
eighteenth century, James Madison noted that the formation of self-interested groups, which he called
factions, was inevitable in any society, as individuals started to work together to protect themselves
from the government. Interest groups and political parties are two of the most easily identified forms of
factions in the United States. These groups are similar in that they are both mediating institutions
responsible for communicating public preferences to the government. They are not themselves
government institutions in a formal sense. Neither is directly mentioned in the U.S. Constitution nor do
they have any real, legal authority to influence policy. But whereas interest groups often work indirectly
to influence our leaders, political parties are organizations that try to directly influence public policy
through its members who seek to win and hold public office. Parties accomplish this by identifying and
aligning sets of issues that are important to voters in the hopes of gaining support during elections; their
positions on these critical issues are often presented in documents known as a party platform (Figure
9.2), which is adopted at each party’s presidential nominating convention every four years. If successful,
a party can create a large enough electoral coalition to gain control of the government. Once in power,
the party is then able to deliver, to its voters and elites, the policy preferences they choose by electing
its partisans to the government. In this respect, parties provide choices to the electorate, something
they are doing that is in such sharp contrast to their opposition. 328 Chapter 9 | Political Parties This
OpenStax book is available for free at http://cnx.org/content/col26739/1.4 Figure 9.2 The party
platform adopted at the first national convention of the Progressive Party in 1912. Among other items,
this platform called for disclosure requirements for campaign contributions, an eight-hour workday, a
federal income tax, and women’s suffrage. You can read the full platform of the Republican Party
(https://openstax.org/l/29gopplatform) and the Democratic Party
(https://openstax.org/l/29demplatform) at their respective websites. Winning elections and
implementing policy would be hard enough in simple political systems, but in a country as complex as
the United States, political parties must take on great responsibilities to win elections and coordinate
behavior across the many local, state, and national governing bodies. Indeed, political differences
between states and local areas can contribute much complexity. If a party stakes out issue positions on
which few people agree and therefore builds too narrow a coalition of voter support, that party may
find itself marginalized. But if the party takes too broad a position on issues, it might find itself in a
situation where the members of the party disagree with one another, making it difficult to pass
legislation, even if the party can secure victory. It should come as no surprise that the story of U.S.
political parties largely mirrors the story of the United States itself. The United States has seen sweeping
changes to its size, its relative power, and its social and demographic composition. These changes have
been mirrored by the political parties as they have sought to shift their coalitions to establish and
maintain power across the nation and as party leadership has changed. As you will learn later, this also
means that the structure and behavior of modern parties largely parallel the social, demographic, and
geographic divisions within the United States today. To understand how this has happened, we look at
the origins of the U.S. party system. Link to Learning Chapter 9 | Political Parties 329 HOW POLITICAL
PARTIES FORMED National political parties as we understand them today did not really exist in the
United States during the early years of the republic. Most politics during the time of the nation’s
founding were local in nature and based on elite politics, limited suffrage (or the ability to vote in
elections), and property ownership. Residents of the various colonies, and later of the various states,
were far more interested in events in their state legislatures than in those occurring at the national level
or later in the nation’s capital. To the extent that national issues did exist, they were largely limited to
collective security efforts to deal with external rivals, such as the British or the French, and with
perceived internal threats, such as conflicts with Native Americans. Soon after the United States
emerged from the Revolutionary War, however, a rift began to emerge between two groups that had
very different views about the future direction of U.S. politics. Thus, from the very beginning of its
history, the United States has had a system of government dominated by two different philosophies.
Federalists, who were largely responsible for drafting and ratifying the U.S. Constitution, generally
favored the idea of a stronger, more centralized republic that had greater control over regulating the
economy.1 Anti-Federalists preferred a more confederate system built on state equality and
autonomy.2 The Federalist faction, led by Alexander Hamilton, largely dominated the government in the
years immediately after the Constitution was ratified. Included in the Federalists was President George
Washington, who was initially against the existence of parties in the United States. When Washington
decided to exit politics and leave office, he warned of the potential negative effects of parties in his
farewell address to the nation, including their potentially divisive nature and the fact that they might
not always focus on the common good but rather on partisan ends. However, members of each faction
quickly realized that they had a vested interest not only in nominating and electing a president who
shared their views, but also in winning other elections. Two loosely affiliated party coalitions, known as
the Federalists and the Democratic-Republicans, soon emerged. The Federalists succeeded in electing
their first leader, John Adams, to the presidency in 1796, only to see the Democratic-Republicans gain
victory under Thomas Jefferson four years later in 1800. 330 Chapter 9 | Political Parties This OpenStax
book is available for free at http://cnx.org/content/col26739/1.4 The “Revolution of 1800”: Uniting the
Executive Branch under One Party When the U.S. Constitution was drafted, its authors were certainly
aware that political parties existed in other countries (like Great Britain), but they hoped to avoid them
in the United States. They felt the importance of states in the U.S. federal structure would make it
difficult for national parties to form. They also hoped that having a college of electors vote for the
executive branch, with the top two vote-getters becoming president and vice president, would
discourage the formation of parties. Their system worked for the first two presidential elections, when
essentially all the electors voted for George Washington to serve as president. But by 1796, the
Federalist and Anti-Federalist camps had organized into electoral coalitions. The Anti-Federalists joined
with many others active in the process to become known as the Democratic-Republicans. The Federalist
John Adams won the Electoral College vote, but his authority was undermined when the vice presidency
went to Democratic-Republican Thomas Jefferson, who finished second. Four years later, the
Democratic-Republicans managed to avoid this outcome by coordinating the electors to vote for their
top two candidates. But when the vote ended in a tie, it was ultimately left to Congress to decide who
would be the third president of the United States (Figure 9.3). Figure 9.3 Thomas Jefferson almost lost
the presidential election of 1800 to his own running mate when a flaw in the design of the Electoral
College led to a tie that had to be resolved by Congress. In an effort to prevent a similar outcome in the
future, Congress and the states voted to ratify the Twelfth Amendment, which went into effect in 1804.
This amendment changed the rules so that the president and vice president would be selected through
separate elections within the Electoral College, and it altered the method that Congress used to fill the
offices in the event that no candidate won a majority. The amendment essentially endorsed the new
party system and helped prevent future controversies. It also served as an early effort by the two parties
to collude to make it harder for an outsider to win the presidency. Does the process of selecting the
executive branch need to be reformed so that the people elect the president and vice president directly,
rather than through the Electoral College? Should the people vote separately on each office rather than
voting for both at the same time? Explain your reasoning. Growing regional tensions eroded the
Federalist Party’s ability to coordinate elites, and it eventually collapsed following its opposition to the
War of 1812.3 The Democratic-Republican Party, on the other hand, eventually divided over whether
national resources should be focused on economic and mercantile development, such as tariffs on
imported goods and government funding of internal improvements like roads and canals, or on
promoting populist issues that would help the “common man,” such as reducing or eliminating state
property requirements that had prevented many men from voting.4 In the election of 1824, numerous
candidates contended for the presidency, all members of the DemocraticMilestone Chapter 9 | Political
Parties 331 Republican Party. Andrew Jackson won more popular votes and more votes in the Electoral
College than any other candidate. However, because he did not win the majority (more than half) of the
available electoral votes, the election was decided by the House of Representatives, as required by the
Twelfth Amendment. The Twelfth Amendment limited the House’s choice to the three candidates with
the greatest number of electoral votes. Thus, Andrew Jackson, with 99 electoral votes, found himself in
competition with only John Quincy Adams, the second place finisher with 84 electoral votes, and
William H. Crawford, who had come in third with 41. The fourth-place finisher, Henry Clay, who was no
longer in contention, had won 37 electoral votes. Clay strongly disliked Jackson, and his ideas on
government support for tariffs and internal improvements were similar to those of Adams. Clay thus
gave his support to Adams, who was chosen on the first ballot. Jackson considered the actions of Clay
and Adams, the son of the Federalist president John Adams, to be an unjust triumph of supporters of
the elite and referred to it as “the corrupt bargain.”5 This marked the beginning of what historians call
the Second Party System (the first parties had been the Federalists and the Jeffersonian Republicans),
with the splitting of the Democratic-Republicans and the formation of two new political parties. One
half, called simply the Democratic Party, was the party of Jackson; it continued to advocate for the
common people by championing westward expansion and opposing a national bank. The branch of the
Democratic-Republicans that believed that the national government should encourage economic
(primarily industrial) development was briefly known as the National Republicans and later became the
Whig Party6 . In the election of 1828, Democrat Andrew Jackson was triumphant. Three times as many
people voted in 1828 as had in 1824, and most cast their ballots for him.7 The formation of the
Democratic Party marked an important shift in U.S. politics. Rather than being built largely to coordinate
elite behavior, the Democratic Party worked to organize the electorate by taking advantage of statelevel laws that had extended suffrage from male property owners to nearly all white men.8 This change
marked the birth of what is often considered the first modern political party in any democracy in the
world.9 It also dramatically changed the way party politics was, and still is, conducted. For one thing,
this new party organization was built to include structures that focused on organizing and mobilizing
voters for elections at all levels of government. The party also perfected an existing spoils system, in
which support for the party during elections was rewarded with jobs in the government bureaucracy
after victory.10 Many of these positions were given to party bosses and their friends. These men were
the leaders of political machines, organizations that secured votes for the party’s candidates or
supported the party in other ways. Perhaps more importantly, this election-focused organization also
sought to maintain power by creating a broader coalition and thereby expanding the range of issues
upon which the party was constructed.11 Each of the two main U.S. political parties today—the
Democrats (https://openstax.org/l/ 29demcratsorg) and the Republicans
(https://openstax.org/l/29gopwebsite) —maintains an extensive website with links to its affiliated
statewide organizations, which in turn often maintain links to the party’s country organizations. By
comparison, here are websites for the Green Party (https://openstax.org/l/29greenparty) and the
Libertarian Party (https://openstax.org/l/29libertarian) that are two other parties in the United States
today. The Democratic Party emphasized personal politics, which focused on building direct
relationships with voters rather than on promoting specific issues. This party dominated national politics
from Andrew Jackson’s presidential victory in 1828 until the mid-1850s, when regional tensions began
to threaten the Link to Learning 332 Chapter 9 | Political Parties This OpenStax book is available for free
at http://cnx.org/content/col26739/1.4 nation’s very existence. The growing power of industrialists,
who preferred greater national authority, combined with increasing tensions between the northern and
southern states over slavery, led to the rise of the Republican Party and its leader Abraham Lincoln in
the election of 1860, while the Democratic Party dominated in the South. Like the Democrats, the
Republicans also began to utilize a mass approach to party design and organization. Their opposition to
the expansion of slavery, and their role in helping to stabilize the Union during Reconstruction, made
them the dominant player in national politics for the next several decades.12 The Democratic and
Republican parties have remained the two dominant players in the U.S. party system since the Civil War
(1861–1865). That does not mean, however, that the system has been stagnant. Every political actor
and every citizen has the ability to determine for him- or herself whether one of the two parties meets
his or her needs and provides an appealing set of policy options, or whether another option is
preferable. At various points in the past 170 years, elites and voters have sought to create alternatives
to the existing party system. Political parties that are formed as alternatives to the Republican and
Democratic parties are known as third parties, or minor parties (Figure 9.4). In 1892, a third party known
as the Populist Party formed in reaction to what its constituents perceived as the domination of U.S.
society by big business and a decline in the power of farmers and rural communities. The Populist Party
called for the regulation of railroads, an income tax, and the popular election of U.S. senators, who at
this time were chosen by state legislatures and not by ordinary voters.13 The party’s candidate in the
1892 elections, James B. Weaver, did not perform as well as the two main party candidates, and, in the
presidential election of 1896, the Populists supported the Democratic candidate William Jennings Bryan.
Bryan lost, and the Populists once again nominated their own presidential candidates in 1900, 1904, and
1908. The party disappeared from the national scene after 1908, but its ideas were similar to those of
the Progressive Party, a new political party created in 1912. Chapter 9 | Political Parties 333 Figure 9.4
Various third parties, also known as minor parties, have appeared in the United States over the years.
Some, like the Socialist Party, still exist in one form or another. Others, like the Anti-Masonic Party,
which wanted to protect the United States from the influence of the Masonic fraternal order and
garnered just under 8 percent of the popular vote in 1832, are gone. In 1912, former Republican
president Theodore Roosevelt attempted to form a third party, known as the Progressive Party, as an
alternative to the more business-minded Republicans. The Progressives sought to correct the many
problems that had arisen as the United States transformed itself from a rural, agricultural nation into an
increasingly urbanized, industrialized country dominated by big business interests. Among the reforms
that the Progressive Party called for in its 1912 platform were women’s suffrage, an eighthour workday,
and workers’ compensation. The party also favored some of the same reforms as the Populist Party,
such as the direct election of U.S. senators and an income tax, although Populists tended to be farmers
while the Progressives were from the middle class. In general, Progressives sought to make government
more responsive to the will of the people and to end political corruption in government. They wished to
break the power of party bosses and political machines, and called upon states to pass laws allowing
voters to vote directly on proposed legislation, propose new laws, and recall from office incompetent or
corrupt elected officials. The Progressive Party largely disappeared after 1916, and most members
returned to the Republican Party.14 The party enjoyed a brief resurgence in 1924, when Robert
“Fighting Bob” La Follette ran unsuccessfully for president under the Progressive banner. 334 Chapter 9
| Political Parties This OpenStax book is available for free at http://cnx.org/content/col26739/1.4 In
1948, two new third parties appeared on the political scene. Henry A. Wallace, a vice president under
Franklin Roosevelt, formed a new Progressive Party, which had little in common with the earlier
Progressive Party. Wallace favored racial desegregation and believed that the United States should have
closer ties to the Soviet Union. Wallace’s campaign was a failure, largely because most people believed
his policies, including national healthcare, were too much like those of communism, and this party also
vanished. The other third party, the States’ Rights Democrats, also known as the Dixiecrats, were white,
southern Democrats who split from the Democratic Party when Harry Truman, who favored civil rights
for African Americans, became the party’s nominee for president. The Dixiecrats opposed all attempts
by the federal government to end segregation, extend voting rights, prohibit discrimination in
employment, or otherwise promote social equality among races.15 They remained a significant party
that threatened Democratic unity throughout the 1950s and 1960s. Other examples of third parties in
the United States include the American Independent Party, the Libertarian Party, United We Stand
America, the Reform Party, and the Green Party. None of these alternatives to the two major political
parties had much success at the national level, and most are no longer viable parties. All faced the same
fate. Formed by charismatic leaders, each championed a relatively narrow set of causes and failed to
gain broad support among the electorate. Once their leaders had been defeated or discredited, the
party structures that were built to contest elections collapsed. And within a few years, most of their
supporters were eventually pulled back into one of the existing parties. To be sure, some of these
parties had an electoral impact. For example, the Progressive Party pulled enough votes away from the
Republicans to hand the 1912 election to the Democrats. Thus, the thirdparty rival’s principal
accomplishment was helping its least-preferred major party win, usually at the short-term expense of
the very issue it championed. In the long run, however, many third parties have brought important
issues to the attention of the major parties, which then incorporated these issues into their platforms.
Understanding why this is the case is an important next step in learning about the issues and strategies
of the modern Republican and Democratic parties. In the next section, we look at why the United States
has historically been dominated by only two political parties. 9.2 The Two-Party System Learning
Objectives By the end of this section, you will be able to: • Describe the effects of winner-take-all
elections • Compare plurality and proportional representation • Describe the institutional, legal, and
social forces that limit the number of parties • Discuss the concepts of party alignment and realignment
One of the cornerstones of a vibrant democracy is citizens’ ability to influence government through
voting. In order for that influence to be meaningful, citizens must send clear signals to their leaders
about what they wish the government to do. It only makes sense, then, that a democracy will benefit if
voters have several clearly differentiated options available to them at the polls on Election Day. Having
these options means voters can select a candidate who more closely represents their own preferences
on the important issues of the day. It also gives individuals who are considering voting a reason to
participate. After all, you are more likely to vote if you care about who wins and who loses. The
existence of two major parties, especially in our present era of strong parties, leads to sharp distinctions
between the candidates and between the party organizations. Why do we have two parties? The twoparty system came into being because the structure of U.S. elections, with one seat tied to a geographic
district, tends to lead to dominance by two major political parties. Even when there are other options on
the ballot, most voters understand that minor parties have no real chance of winning even a single
office. Hence, they vote for candidates of the two major Chapter 9 | Political Parties 335 parties in order
to support a potential winner. Of the 535 members of the House and Senate, only a handful identify as
something other than Republican or Democrat. Third parties have fared no better in presidential
elections. No third-party candidate has ever won the presidency. Some historians or political scientists
might consider Abraham Lincoln to have been such a candidate, but in 1860, the Republicans were a
major party that had subsumed members of earlier parties, such as the Whig Party, and they were the
only major party other than the Democratic Party. ELECTION RULES AND THE TWO-PARTY SYSTEM A
number of reasons have been suggested to explain why the structure of U.S. elections has resulted in a
two-party system. Most of the blame has been placed on the process used to select its representatives.
First, most elections at the state and national levels are winner-take-all: The candidate who receives the
greatest overall number of votes wins. Winner-take-all elections with one representative elected for one
geographic district allow voters to develop a personal relationship with “their” representative to the
government. They know exactly whom to blame, or thank, for the actions of that government. But these
elections also tend to limit the number of people who run for office. Otherwise-qualified candidates
might not stand for election if they feel the incumbent or another candidate has an early advantage in
the race. And since voters do not like to waste votes, third parties must convince voters they have a real
chance of winning races before voters will take them seriously. This is a tall order given the vast
resources and mobilization tools available to the existing parties, especially if an incumbent is one of the
competitors. In turn, the likelihood that third-party challengers will lose an election bid makes it more
difficult to raise funds to support later attempts.16 Winner-take-all systems of electing candidates to
office, which exist in several countries other than the United States, require that the winner receive
either the majority of votes or a plurality of the votes. U.S. elections are based on plurality voting.
Plurality voting, commonly referred to as first-past-the-post, is based on the principle that the individual
candidate with the most votes wins, whether or not he or she gains a majority (51 percent or greater) of
the total votes cast. For instance, Abraham Lincoln won the presidency in 1860 even though he clearly
lacked majority support given the number of candidates in the race. In 1860, four candidates competed
for the presidency: Lincoln, a Republican; two Democrats, one from the northern wing of the party and
one from the southern wing; and a member of the newly formed Constitutional Union Party, a southern
party that wished to prevent the nation from dividing over the issue of slavery. Votes were split among
all four parties, and Lincoln became president with only 40 percent of the vote, not a majority of votes
cast but more than any of the other three candidates had received, and enough to give him a majority in
the Electoral College, the body that ultimately decides presidential elections. Plurality voting has been
justified as the simplest and most cost-effective method for identifying a victor in a democracy. A single
election can be held on a single day, and the victor of the competition is easily selected. On the other
hand, systems in which people vote for a single candidate in an individual district often cost more
money because drawing district lines and registering voters according to district is often expensive and
cumbersome.17 In a system in which individual candidates compete for individual seats representing
unique geographic districts, a candidate must receive a fairly large number of votes in order to win. A
political party that appeals to only a small percentage of voters will always lose to a party that is more
popular.18 Because second-place (or lower) finishers will receive no reward for their efforts, those
parties that do not attract enough supporters to finish first at least some of the time will eventually
disappear because their supporters realize they have no hope of achieving success at the polls.19 The
failure of third parties to win and the possibility that they will draw votes away from the party the voter
had favored before—resulting in a win for the party the voter liked least—makes people hesitant to
vote for the third party’s candidates a second time. This has been the fate of all U.S. third parties—the
Populist Party, the Progressives, the Dixiecrats, the Reform Party, and others. In a proportional electoral
system, however, parties advertise who is on their candidate list and voters pick a party. Then,
legislative seats are doled out to the parties based on the proportion of support each 336 Chapter 9 |
Political Parties This OpenStax book is available for free at http://cnx.org/content/col26739/1.4 party
receives. While the Green Party in the United States might not win a single congressional seat in some
years thanks to plurality voting, in a proportional system, it stands a chance to get a few seats in the
legislature regardless. For example, assume the Green Party gets 7 percent of the vote. In the United
States, 7 percent will never be enough to win a single seat, shutting the Green candidates out of
Congress entirely, whereas in a proportional system, the Green Party will get 7 percent of the total
number of legislative seats available. Hence, it could get a foothold for its issues and perhaps increase its
support over time. But with plurality voting, it doesn’t stand a chance. Third parties, often born of
frustration with the current system, attract supporters from one or both of the existing parties during an
election but fail to attract enough votes to win. After the election is over, supporters experience
remorse when their least-favorite candidate wins instead. For example, in the 2000 election, Ralph
Nader ran for president as the candidate of the Green Party. Nader, a longtime consumer activist
concerned with environmental issues and social justice, attracted many votes from people who usually
voted for Democratic candidates. This has caused some to claim that Democratic nominee Al Gore lost
the 2000 election to Republican George W. Bush, because Nader won Democratic votes in Florida that
might otherwise have gone to Gore (Figure 9.5).20 Figure 9.5 Ralph Nader, a longtime consumer
advocate and crusader for social justice and the environment, campaigned as an independent in 2008
(a). However, in 2000, he ran for the presidency as the Green Party candidate. He received votes from
many Democrats, and some analysts claim Nader’s campaign cost Al Gore the presidency—an ironic
twist for a politician who would come to be known primarily for his environmental activism, even
winning the Nobel Peace Prize in 2007 (b) for his efforts to inform the public about climate change.
(credit a: modification of work by “Mely-o”/Flikr”; credit b: modification of work by
“kangotraveler”/Flickr) Abandoning plurality voting, even if the winner-take-all election were kept,
would almost certainly increase the number of parties from which voters could choose. The easiest
switch would be to a majoritarian voting scheme, in which a candidate wins only if he or she enjoys the
support of a majority of voters. If no candidate wins a majority in the first round of voting, a run-off
election is held among the top contenders. Some states conduct their primary elections within the two
major political parties in this way. A second way to increase the number of parties in the U.S. system is
to abandon the winner-take-all approach. Rather than allowing voters to pick their representatives
directly, many democracies have chosen to have voters pick their preferred party and allow the party to
select the individuals who serve in government. The argument for this method is that it is ultimately the
party and not the individual who will influence policy. Under this model of proportional representation,
legislative seats are allocated to competing parties based on the total share of votes they receive in the
election. As a result, any given election can have multiple winners, and voters who might prefer a
smaller party over a major one have a Chapter 9 | Political Parties 337 chance to be represented in
government (Figure 9.6). Figure 9.6 While a U.S. ballot (a) for first-past-the-post elections features
candidates’ names, the ballots of proportional representation countries list the parties. On this Russian
ballot (b), the voter is offered a choice of Social Democratic, Nationalist, Socialist, and Communist
parties, among others. One possible way to implement proportional representation in the United States
is to allocate legislative seats based on the national level of support for each party’s presidential
candidate, rather than on the results of individual races. If this method had been used in the 1996
elections, 8 percent of the seats in Congress would have gone to Ross Perot’s Reform Party because he
won 8 percent of the votes cast. Even though Perot himself lost, his supporters would have been
rewarded for their efforts with representatives who had a real voice in government. And Perot’s party’s
chances of survival would have greatly increased. Electoral rules are probably not the only reason the
United States has a two-party system. We need only look at the number of parties in the British or
Canadian systems, both of which are winner-take-all plurality systems like that in the United States, to
see that it is possible to have more than two parties while still directly electing representatives. The twoparty system is also rooted in U.S. history. The first parties, the Federalists and the Jeffersonian
Republicans, disagreed about how much power should be given to the federal government, and
differences over other important issues further strengthened this divide. Over time, these parties
evolved into others by inheriting, for the most part, the general ideological positions and constituents of
their predecessors, but no more than two major parties ever formed. Instead of parties arising based on
region or ethnicity, various regions and ethnic groups sought a place in one of the two major parties.
Scholars of voting behavior have also suggested at least three other characteristics of the U.S. system
that are likely to influence party outcomes: the Electoral College, demobilized ethnicity, and campaign
and election laws. First, the United States has a presidential system in which the winner is selected not
directly by the popular vote but indirectly by a group of electors known collectively as the Electoral
College. The winner-take-all system also applies in the Electoral College. In all but two states (Maine and
Nebraska), the total of the state’s electoral votes go to the candidate who wins the plurality of the
popular vote in that 338 Chapter 9 | Political Parties This OpenStax book is available for free at
http://cnx.org/content/col26739/1.4 state. Even if a new, third party is able to win the support of a lot
of voters, it must be able to do so in several states in order to win enough electoral votes to have a
chance of winning the presidency.21 Besides the existence of the Electoral College, political scientist
Gary W. Cox has also suggested that the relative prosperity of the United States and the relative unity of
its citizens have prevented the formation of “large dissenting groups” that might give support to third
parties.22 This is similar to the argument that the United States does not have viable third parties,
because none of its regions is dominated by mobilized ethnic minorities that have created political
parties in order to defend and to address concerns solely of interest to that ethnic group. Such parties
are common in other countries. Finally, party success is strongly influenced by local election laws.
Someone has to write the rules that govern elections, and those rules help to determine outcomes. In
the United States, such rules have been written to make it easy for existing parties to secure a spot for
their candidates in future elections. But some states create significant burdens for candidates who wish
to run as independents or who choose to represent new parties. For example, one common practice is
to require a candidate who does not have the support of a major party to ask registered voters to sign a
petition. Sometimes, thousands of signatures are required before a candidate’s name can be placed on
the ballot (Figure 9.7), but a small third party that does have large numbers of supporters in some states
may not be able to secure enough signatures for this to happen.23 Figure 9.7 Costa Constantinides
(right), while campaigning in 2013 to represent the 22nd District on the New York City Council, said,
“Few things are more important to a campaign than the petition process to get on the ballot. We were
so pumped up to get started that we went out at 12:01 a.m. on June 4 to start collecting signatures right
away!” Constantinides won the election later that year. (credit: modification of work by Costa
Constantinides) Visit Fair Vote (https://openstax.org/l/29fairvoteweb) for a discussion of ballot access
laws across the country. Given the obstacles to the formation of third parties, it is unlikely that serious
challenges to the U.S. twoparty system will emerge. But this does not mean that we should view it as
entirely stable either. The U.S. party system is technically a loose organization of fifty different state
parties and has undergone several considerable changes since its initial consolidation after the Civil War.
Third-party movements may have Link to Learning Chapter 9 | Political Parties 339 played a role in some
of these changes, but all resulted in a shifting of party loyalties among the U.S. electorate. CRITICAL
ELECTIONS AND REALIGNMENT Political parties exist for the purpose of winning elections in order to
influence public policy. This requires them to build coalitions across a wide range of voters who share
similar preferences. Since most U.S. voters identify as moderates,24 the historical tendency has been for
the two parties to compete for “the middle” while also trying to mobilize their more loyal bases. If
voters’ preferences remained stable for long periods of time, and if both parties did a good job of
competing for their votes, we could expect Republicans and Democrats to be reasonably competitive in
any given election. Election outcomes would probably be based on the way voters compared the parties
on the most important events of the day rather than on electoral strategy. There are many reasons we
would be wrong in these expectations, however. First, the electorate isn’t entirely stable. Each
generation of voters has been a bit different from the last. Over time, the United States has become
more socially liberal, especially on topics related to race and gender, and Millennials—those aged 21–
37—are more liberal than members of older generations.25 The electorate’s economic preferences
have changed, and different social groups are likely to become more engaged in politics now than they
did in the past. Surveys conducted in 2016, for example, revealed that candidates’ religion is less
important to voters than it once was. Also, as young Latinos reach voting age, they seem more inclined
to vote than do their parents, which may raise the traditionally low voting rates among this ethnic
group.26 Internal population shifts and displacements have also occurred, as various regions have taken
their turn experiencing economic growth or stagnation, and as new waves of immigrants have come to
U.S. shores. Additionally, the major parties have not always been unified in their approach to contesting
elections. While we think of both Congress and the presidency as national offices, the reality is that
congressional elections are sometimes more like local elections. Voters may reflect on their preferences
for national policy when deciding whom to send to the Senate or the House of Representatives, but they
are very likely to view national policy in the context of its effects on their area, their family, or
themselves, not based on what is happening to the country as a whole. For example, while many voters
want to reduce the federal budget, those over sixty-five are particularly concerned that no cuts to the
Medicare program be made.27 Onethird of those polled reported that “senior’s issues” were most
important to them when voting for national officeholders.28 If they hope to keep their jobs, elected
officials must thus be sensitive to preferences in their home constituencies as well as the preferences of
their national party. Finally, it sometimes happens that over a series of elections, parties may be unable
or unwilling to adapt their positions to broader socio-demographic or economic forces. Parties need to
be aware when society changes. If leaders refuse to recognize that public opinion has changed, the
party is unlikely to win in the next election. For example, people who describe themselves as evangelical
Christians are an important Republican constituency; they are also strongly opposed to abortion.29
Thus, even though the majority of U.S. adults believe abortion should be legal in at least some instances,
such as when a pregnancy is the result of rape or incest, or threatens the life of the mother, the position
of many Republican presidential candidates in 2016 was to oppose abortion in all cases.30 As a result,
many women view the Republican Party as unsympathetic to their interests and are more likely to
support Democratic candidates.31 Similarly (or simultaneously), groups that have felt that the party has
served their causes in the past may decide to look elsewhere if they feel their needs are no longer being
met. Either way, the party system will be upended as a result of a party realignment, or a shifting of
party allegiances within the electorate (Table 9.1).32 340 Chapter 9 | Political Parties This OpenStax
book is available for free at http://cnx.org/content/col26739/1.4 Periods of Party Dominance and
Realignment Era Party Systems and Realignments 1796–1824 First Party System: Federalists (urban
elites, southern planters, New England) oppose Democratic-Republicans (rural, small farmers and
artisans, the South and the West). 1828–1856 Second Party System: Democrats (the South, cities,
farmers and artisans, immigrants) oppose Whigs (former Federalists, the North, middle class, nativeborn Americans). 1860–1892 Third Party System: Republicans (former Whigs plus African Americans)
control the presidency. Only one Democrat, Grover Cleveland, is elected president (1884, 1892). 1896–
1932 Fourth Party System: Republicans control the presidency. Only one Democrat, Woodrow Wilson, is
elected president (1912, 1916). Challenges to major parties are raised by Populists and Progressives.
1932–1964 Fifth Party System. Democrats control the presidency. Only one Republican, Dwight
Eisenhower, is elected president (1952, 1956). Major party realignment as African Americans become
part of the Democratic coalition. 1964–present Sixth Party System. No one party controls the
presidency. Ongoing realignment as southern whites and many northern members of the working class
begin to vote for Republicans. Latinos and Asians immigrate, most of whom vote for Democrats. Table
9.1 There have been six distinctive periods in U.S. history when new political parties have emerged,
control of the presidency has shifted from one party to another, or significant changes in a party’s
makeup have occurred. One of the best-known party realignments occurred when Democrats moved to
include African Americans and other minorities into their national coalition during the Great Depression.
After the Civil War, Republicans, the party of Lincoln, were viewed as the party that had freed the slaves.
Their efforts to provide blacks with greater legal rights earned them the support of African Americans in
both the South, where they were newly enfranchised, and the Northeast. When the Democrats, the
party of the Confederacy, lost control of the South after the Civil War, Republicans ruled the region.
However, the Democrats regained control of the South after the removal of the Union army in 1877.
Democrats had largely supported slavery before the Civil War, and they opposed postwar efforts to
integrate African Americans into society after they were liberated. In addition, Democrats in the North
and Midwest drew their greatest support from labor union members and immigrants who viewed
African Americans as competitors for jobs and government resources, and who thus tended to oppose
the extension of rights to African Americans as much as their southern counterparts did.33 While the
Democrats’ opposition to civil rights may have provided regional advantages in southern or urban
elections, it was largely disastrous for national politics. From 1868 to 1931, Democratic candidates won
just four of sixteen presidential elections. Two of these victories can be explained as a result of the
spoiler effect of the Progressive Party in 1912 and then Woodrow Wilson’s reelection during World War
I in 1916. This rather-dismal success rate suggested that a change in the governing coalition would be
needed if the party were to have a chance at once again becoming a player on the national level. That
change began with the 1932 presidential campaign of Franklin Delano Roosevelt. FDR determined that
his best path toward victory was to create a new coalition based not on region or ethnicity, but on the
suffering of those hurt the most during the Great Depression. This alignment sought to bring African
American voters in as a means of shoring up support in major urban areas and the Midwest, where
many southern blacks had migrated in the decades after the Civil War in search of jobs and better
education for their children, as well as to avoid many of the legal restrictions placed on them in the
South. Roosevelt accomplished this realignment by promising assistance to those hurt most by the
Depression, including African Americans. Chapter 9 | Political Parties 341 The strategy worked.
Roosevelt won the election with almost 58 percent of the popular vote and 472 Electoral College votes,
compared to incumbent Herbert Hoover’s 59. The 1932 election is considered an example of a critical
election, one that represents a sudden, clear, and long-term shift in voter allegiances. After this election,
the political parties were largely identified as being divided by differences in their members’ socioeconomic status. Those who favor stability of the current political and economic system tend to vote
Republican, whereas those who would most benefit from changing the system usually favor Democratic
candidates. Based on this alignment, the Democratic Party won the next five consecutive presidential
elections and was able to build a political machine that dominated Congress into the 1990s, including
holding an uninterrupted majority in the House of Representatives from 1954 until 1994. The
realignment of the parties did have consequences for Democrats. African Americans became an
increasingly important part of the Democratic coalition in the 1940s through the 1960s, as the party
took steps to support civil rights.34 Most changes were limited to the state level at first, but as civil
rights reform moved to the national stage, rifts between northern and southern Democrats began to
emerge.35 Southern Democrats became increasingly convinced that national efforts to provide social
welfare and encourage racial integration were violating state sovereignty and social norms. By the
1970s, many had begun to shift their allegiance to the Republican Party, whose pro-business wing
shared their opposition to the growing encroachment of the national government into what they viewed
as state and local matters.36 Almost fifty years after it had begun, the realignment of the two political
parties resulted in the flipping of post-Civil War allegiances, with urban areas and the Northeast now
solidly Democratic, and the South and rural areas overwhelmingly voting Republican. The result today is
a political system that provides Republicans with considerable advantages in rural areas and most parts
of the Deep South.37 Democrats dominate urban politics and those parts of the South, known as the
Black Belt, where the majority of residents are African American. 9.3 The Shape of Modern Political
Parties Learning Objectives By the end of this section, you will be able to: • Differentiate between the
party in the electorate and the party organization • Discuss the importance of voting in a political party
organization • Describe party organization at the county, state, and national levels • Compare the
perspectives of the party in government and the party in the electorate We have discussed the two
major political parties in the United States, how they formed, and some of the smaller parties that have
challenged their dominance over time. However, what exactly do political parties do? If the purpose of
political parties is to work together to create and implement policies by winning elections, how do they
accomplish this task, and who actually participates in the process? The answer was fairly straightforward
in the early days of the republic when parties were little more than electoral coalitions of like-minded,
elite politicians. But improvements in strategy and changes in the electorate forced the parties to
become far more complex organizations that operate on several levels in the U.S. political arena.
Modern political parties consist of three components identified by political scientist V. O. Key: the party
in the electorate (the voters); the party organization (which helps to coordinate everything the party
does in its quest for office); and the party in office (the office holders). To understand how these various
elements work together, we begin by thinking about a key first step in influencing policy in any
democracy: winning elections. THE PARTY-IN-THE-ELECTORATE A key fact about the U.S. political party
system is that it’s all about the votes. If voters do not show up to 342 Chapter 9 | Political Parties This
OpenStax book is available for free at http://cnx.org/content/col26739/1.4 vote for a party’s candidates
on Election Day, the party has no chance of gaining office and implementing its preferred policies. As we
have seen, for much of their history, the two parties have been adapting to changes in the size,
composition, and preferences of the U.S. electorate. It only makes sense, then, that parties have found
it in their interest to build a permanent and stable presence among the voters. By fostering a sense of
loyalty, a party can insulate itself from changes in the system and improve its odds of winning elections.
The party-in-the-electorate are those members of the voting public who consider themselves to be part
of a political party and/or who consistently prefer the candidates of one party over the other. What it
means to be part of a party depends on where a voter lives and how much he or she chooses to
participate in politics. At its most basic level, being a member of the party-in-the-electorate simply
means a voter is more likely to voice support for a party. These voters are often called party identifiers,
since they usually represent themselves in public as being members of a party, and they may attend
some party events or functions. Party identifiers are also more likely to provide financial support for the
candidates of their party during election season. This does not mean self-identified Democrats will
support all the party’s positions or candidates, but it does mean that, on the whole, they feel their
wants or needs are more likely to be met if the Democratic Party is successful. Party identifiers make up
the majority of the voting public. Gallup, the polling agency, has been collecting data on voter
preferences for the past several decades. Its research suggests that historically, over half of American
adults have called themselves “Republican” or “Democrat” when asked how they identify themselves
politically (Figure 9.8). Even among self-proclaimed independents, the overwhelming majority claim to
lean in the direction of one party or the other, suggesting they behave as if they identified with a party
during elections even if they preferred not to publicly pick a side. Partisan support is so strong that, in a
poll conducted from August 5 to August 9, 2015, about 88 percent of respondents said they either
identified with or, if they were independents, at least leaned toward one of the major political
parties.38 Thus, in a poll conducted in January 2016, even though about 42 percent of respondents said
they were independent, this does not mean that they are not, in fact, more likely to favor one party over
the other.39 Chapter 9 | Political Parties 343 Figure 9.8 As the chart reveals, generation affects party
identification. Millennials (aged 21–37) are more likely to identify as or lean towards the Democratic
Party and less likely to favor Republicans than are their Baby Boomer parents and grandparents (born
between 1946 and 1964). Strictly speaking, party identification is not quite the same thing as party
membership. People may call themselves Republicans or Democrats without being registered as a
member of the party, and the Republican and Democratic parties do not require individuals to join their
formal organization in the same way that parties in some other countries do. Many states require voters
to declare a party affiliation before participating in primaries, but primary participation is irregular and
infrequent, and a voter may change his or her identity long before changing party registration. For most
voters, party identification is informal at best and often matters only in the weeks before an election. It
does matter, however, because party identification guides some voters, who may know little about a
particular issue or candidate, in casting their ballots. If, for example, someone thinks of him- or herself
as a Republican and always votes Republican, he or she will not be confused when faced with a
candidate, perhaps in a local or county election, whose name is unfamiliar. If the candidate is a
Republican, the voter will likely cast a ballot for him or her. Party ties can manifest in other ways as well.
The actual act of registering to vote and selecting a party reinforces party loyalty. Moreover, while
pundits and scholars often deride voters who blindly vote their party, the selection of a party in the first
place can be based on issue positions and ideology. In that regard, voting your party on Election Day is
not a blind act—it is a shortcut based on issue positions. 344 Chapter 9 | Political Parties This OpenStax
book is available for free at http://cnx.org/content/col26739/1.4 THE PARTY ORGANIZATION A
significant subset of American voters views their party identification as something far beyond simply a
shortcut to voting. These individuals get more energized by the political process and have chosen to
become more active in the life of political parties. They are part of what is known as the party
organization. The party organization is the formal structure of the political party, and its active members
are responsible for coordinating party behavior and supporting party candidates. It is a vital component
of any successful party because it bears most of the responsibility for building and maintaining the party
“brand.” It also plays a key role in helping select, and elect, candidates for public office. Local
Organizations Since winning elections is the first goal of the political party, it makes sense that the
formal party organization mirrors the local-state-federal structure of the U.S. political system. While the
lowest level of party organization is technically the precinct, many of the operational responsibilities for
local elections fall upon the county-level organization. The county-level organization is in many ways the
workhorse of the party system, especially around election time. This level of organization frequently
takes on many of the most basic responsibilities of a democratic system, including identifying and
mobilizing potential voters and donors, identifying and training potential candidates for public office,
and recruiting new members for the party. County organizations are also often responsible for finding
rank and file members to serve as volunteers on Election Day, either as officials responsible for
operating the polls or as monitors responsible for ensuring that elections are conducted honestly and
fairly. They may also hold regular meetings to provide members the opportunity to meet potential
candidates and coordinate strategy (Figure 9.9). Of course, all this is voluntary and relies on dedicated
party members being willing to pitch in to run the party. Figure 9.9 Political parties are bottom-up
structures, with lower levels often responsible for selecting delegates to higher-level offices or
conventions. Chapter 9 | Political Parties 345 State Organizations Most of the county organizations’
formal efforts are devoted to supporting party candidates running for county and city offices. But a fair
amount of political power is held by individuals in statewide office or in state-level legislative or judicial
bodies. While the county-level offices may be active in these local competitions, most of the
coordination for them will take place in the state-level organizations. Like their more local counterparts,
state-level organizations are responsible for key party functions, such as statewide candidate
recruitment and campaign mobilization. Most of their efforts focus on electing high-ranking officials
such as the governor or occupants of other statewide offices (e.g., the state’s treasurer or attorney
general) as well as candidates to represent the state and its residents in the U.S. Senate and the U.S.
House of Representatives. The greater value of state- and national-level offices requires state
organizations to take on several key responsibilities in the life of the party. Visit the following Republican
(https://openstax.org/l/29iowagoporg) and Democratic (https://openstax.org/l/29ridemocrats) sites to
see what party organizations look like on the local level. Although these sites are for different parties in
different parts of the country, they both inform visitors of local party events, help people volunteer to
work for the party, and provide a convenient means of contributing to the party. First, state-level
organizations usually accept greater fundraising responsibilities than do their local counterparts.
Statewide races and races for national office have become increasingly expensive in recent years. The
average cost of a successful House campaign was $1.2 million in 2014; for Senate races, it was $8.6
million.40 While individual candidates are responsible for funding and running their own races, it is
typically up to the state-level organization to coordinate giving across multiple races and to develop the
staffing expertise that these candidates will draw upon at election time. State organizations are also
responsible for creating a sense of unity among members of the state party. Building unity can be very
important as the party transitions from sometimes-contentious nomination battles to the all-important
general election. The state organization uses several key tools to get its members working together
towards a common goal. First, it helps the party’s candidates prepare for state primary elections or
caucuses that allow voters to choose a nominee to run for public office at either the state or national
level. Caucuses are a form of town hall meeting at which voters in a precinct get together to voice their
preferences, rather than voting individually throughout the day (Figure 9.10). Link to Learning 346
Chapter 9 | Political Parties This OpenStax book is available for free at
http://cnx.org/content/col26739/1.4 Figure 9.10 Caucus-goers gather at a Democratic precinct caucus
on January 3, 2008, in Iowa City, Iowa. Caucuses are held every two years in more than 1650 Iowa
precincts. Second, the state organization is also responsible for drafting a state platform that serves as a
policy guide for partisans who are eventually selected to public office. These platforms are usually the
result of a negotiation between the various coalitions within the party and are designed to ensure that
everyone in the party will receive some benefits if their candidates win the election. Finally, state
organizations hold a statewide convention at which delegates from the various county organizations
come together to discuss the needs of their areas. The state conventions are also responsible for
selecting delegates to the national convention. National Party Organization The local and state-level
party organizations are the workhorses of the political process. They take on most of the responsibility
for party activities and are easily the most active participants in the party formation and electoral
processes. They are also largely invisible to most voters. The average citizen knows very little of the local
party’s behavior unless there is a phone call or a knock on the door in the days or weeks before an
election. The same is largely true of the activities of the state-level party. Typically, the only people who
notice are those who are already actively engaged in politics or are being targeted for donations. But
most people are aware of the presence and activity of the national party organizations for several
reasons. First, many Americans, especially young people, are more interested in the topics discussed at
the national level than at the state or local level. According to John Green of the Ray C. Bliss Institute of
Applied Politics, “Local elections tend to be about things like sewers, and roads and police protection—
which are not as dramatic an issue as same-sex marriage or global warming or international affairs.”41
Presidential elections and the behavior of the U.S. Congress are also far more likely to make the news
broadcasts than the activities of county commissioners, and the national-level party organization is
mostly responsible for coordinating the activities of participants at this level. The national party is a
fundraising army for presidential candidates and also serves a key role in trying to coordinate and direct
the efforts of the House and Senate. For this reason, its leadership is far more likely to become visible to
media consumers, whether they intend to vote or not. A second reason for the prominence of the
national organization is that it usually coordinates the grandest spectacles in the life of a political party.
Most voters are never aware of the numerous county-level meetings or coordinating activities. Primary
elections, one of the most important events to take place at the state level, have a much lower turnout
than the nationwide general election. In 2012, for example, only one-third of the eligible voters in New
Hampshire voted in the state’s primary, one of the earliest and thus most important in the nation;
however, 70 percent of eligible voters in the state voted in the general election in November 2012.42
People may see or read an occasional story about the meetings of the state Chapter 9 | Political Parties
347 committees or convention but pay little attention. But the national conventions, organized and
sponsored by the national-level party, can dominate the national discussion for several weeks in late
summer, a time when the major media outlets are often searching for news. These conventions are the
definition of a media circus at which high-ranking politicians, party elites, and sometimes celebrities,
such as actor/director Clint Eastwood (Figure 9.11), along with individuals many consider to be the
future leaders of the party are brought before the public so the party can make its best case for being
the one to direct the future of the country.43 National party conventions culminate in the formal
nomination of the party nominees for the offices of president and vice president, and they mark the
official beginning of the presidential competition between the two parties. Figure 9.11 In August 2012,
Clint Eastwood—actor, director, and former mayor of Carmel-by-the-Sea, California—spoke at the
Republican National Convention accompanied by an empty chair representing the Democratic
incumbent president Barack Obama. In the past, national conventions were often the sites of high
drama and political intrigue. As late as 1968, the identities of the presidential and/or vice-presidential
nominees were still unknown to the general public when the convention opened. It was also common
for groups protesting key events and issues of the day to try to raise their profile by using the
conventions to gain the media spotlight. National media outlets would provide “gavel to gavel” coverage
of the conventions, and the relatively limited number of national broadcast channels meant most
viewers were essentially forced to choose between following the conventions or checking out of the
media altogether. Much has changed since the 1960s, however, and between 1960 and 2004,
viewership of both the Democratic National Convention and the Republican National Convention had
declined by half.44 National conventions are not the spectacles they once were, and this fact is almost
certainly having an impact on the profile of the national party organization. Both parties have come to
recognize the value of the convention as a medium through which they can communicate to the average
viewer. To ensure that they are viewed in the best possible light, the parties have worked hard to turn
the public face of the convention into a highly sanitized, highly orchestrated media event. Speakers are
often required to have their speeches prescreened to ensure that they do not deviate from the party
line or run the risk of embarrassing the eventual nominee—whose name has often been known by all
for several months. And while protests still happen, party organizations have becoming increasingly
adept at keeping protesters away from the convention sites, arguing that safety and security are more
important than First Amendment rights to speech and peaceable assembly. For example, protestors
were kept behind concrete barriers and fences at the Democratic National Convention in 2004.45 With
the advent of cable TV news and the growth of internet blogging, the major news outlets have found it
unnecessary to provide the same level of coverage they once did. Between 1976 and 1996, ABC and 348
Chapter 9 | Political Parties This OpenStax book is available for free at
http://cnx.org/content/col26739/1.4 CBS cut their coverage of the nominating conventions from more
than fifty hours to only five. NBC cut its coverage to fewer than five hours.46 One reason may be that
the outcome of nominating conventions are also typically known in advance, meaning there is no
drama. Today, the nominee’s acceptance speech is expected to be no longer than an hour, so it will not
take up more than one block of prime-time TV programming. This is not to say the national conventions
are no longer important, or that the national party organizations are becoming less relevant. The
conventions, and the organizations that run them, still contribute heavily to a wide range of key
decisions in the life of both parties. The national party platform is formally adopted at the convention,
as are the key elements of the strategy for contesting the national campaign. And even though the
media is paying less attention, key insiders and major donors often use the convention as a way of
gauging the strength of the party and its ability to effectively organize and coordinate its members. They
are also paying close attention to the rising stars who are given time at the convention’s podium, to see
which are able to connect with the party faithful. Most observers credit Barack Obama’s speech at the
2004 Democratic National Convention with bringing him to national prominence.47 Conventions and
Trial Balloons While both political parties use conventions to help win the current elections, they also
use them as a way of elevating local politicians to the national spotlight. This has been particularly true
for the Democratic Party. In 1988, the Democrats tapped Arkansas governor Bill Clinton to introduce
their nominee Michael Dukakis at the convention. Clinton’s speech was lampooned for its length and
lack of focus, but it served to get his name in front of Democratic voters. Four years later, Clinton was
able to leverage this national exposure to help his own presidential campaign. The pattern was repeated
when Illinois state senator Barack Obama gave the keynote address at the 2004 convention (Figure
9.12). Although he was only a candidate for the U.S. Senate at the time, his address caught the attention
of the Democratic establishment and ultimately led to his emergence as a viable presidential candidate
just four years later. Figure 9.12 Barack Obama gives his “Two Americas” speech at the Democratic
National Convention in Boston in July 2004. At the time, he was an Illinois state senator running for the
U.S. Senate. Should the media devote more attention to national conventions? Would this help voters
choose the candidate they want to vote for? Insider Perspective Chapter 9 | Political Parties 349 Bill
Clinton’s lengthy nomination speech (https://openstax.org/l/29billclinnomsp) in 1988 was much
derided, but served the purpose of providing national exposure to a state governor. Barack Obama’s
inspirational speech (https://openstax.org/l/29barobanomsp) at the 2004 national convention resulted
in immediate speculation as to his wider political aspirations. THE PARTY-IN-GOVERNMENT One of the
first challenges facing the party-in-government, or the party identifiers who have been elected or
appointed to hold public office, is to achieve their policy goals. The means to do this is chosen in
meetings of the two major parties; Republican meetings are called party conferences and Democrat
meetings are called party caucuses. Members of each party meet in these closed sessions and discuss
what items to place on the legislative agenda and make decisions about which party members should
serve on the committees that draft proposed laws. Party members also elect the leaders of their
respective parties in the House and the Senate, and their party whips. Leaders serve as party managers
and are the highestranking members of the party in each chamber of Congress. The party whip ensures
that members are present when a piece of legislation is to be voted on and directs them how to vote.
The whip is the secondhighest ranking member of the party in each chamber. Thus, both the
Republicans and the Democrats have a leader and a whip in the House, and a leader and a whip in the
Senate. The leader and whip of the party that holds the majority of seats in each house are known as
the majority leader and the majority whip. The leader and whip of the party with fewer seats are called
the minority leader and the minority whip. The party that controls the majority of seats in the House of
Representatives also elects someone to serve as Speaker of the House. People elected to Congress as
independents (that is, not members of either the Republican or Democratic parties) must choose a party
to conference or caucus with. For example, Senator Bernie Sanders of Vermont, who originally ran for
Senate as an independent candidate, caucuses with the Democrats and ran for the presidency as a
Democrat. He returned to the Senate in 2017 as an independent.48 The political parties in government
must represent their parties and the entire country at the same time. One way they do this is by
creating separate governing and party structures in the legislature, even though these are run by the
same people. Check out some of the more important leadership organizations and their partisan
counterparts in the House of Representatives (https://openstax.org/l/ 29hofreporg) and the Senate
(https://openstax.org/l/29senateorga) leadership. Link to Learning Link to Learning 350 Chapter 9 |
Political Parties This OpenStax book is available for free at http://cnx.org/content/col26739/1.4 Party
Organization from the Inside Interested in a cool summer job? Want to actually make a difference in
your community? Consider an internship at the Democratic National Committee (DNC) or Republican
National Committee (RNC). Both organizations offer internship programs for college students who want
hands-on experience working in community outreach and grassroots organizing. While many internship
opportunities are based at the national headquarters in Washington, DC, openings may exist within
state party organizations. Internship positions can be very competitive; most applicants are juniors or
seniors with high grade-point averages and strong recommendations from their faculty. Successful
applicants get an inside view of government, build a great professional network, and have the
opportunity to make a real difference in the lives of their friends and families. Visit the DNC or RNC
website and find out what it takes to be an intern. While there, also check out the state party
organization. Is there a local leader you feel you could work for? Are any upcoming events scheduled in
your state? One problem facing the party-in-government relates to the design of the country’s political
system. The U.S. government is based on a complex principle of separation of powers, with power
divided among the executive, legislative, and judiciary branches. The system is further complicated by
federalism, which relegates some powers to the states, which also have separation of powers. This
complexity creates a number of problems for maintaining party unity. The biggest is that each level and
unit of government has different constituencies that the office holder must satisfy. The person elected
to the White House is more beholden to the national party organization than are members of the House
or Senate, because members of Congress must be reelected by voters in very different states, each with
its own state-level and countylevel parties. Some of this complexity is eased for the party that holds the
executive branch of government. Executive offices are typically more visible to the voters than the
legislature, in no small part because a single person holds the office. Voters are more likely to show up
at the polls and vote if they feel strongly about the candidate running for president or governor, but
they are also more likely to hold that person accountable for the government’s failures.49 Members of
the legislature from the executive’s party are under a great deal of pressure to make the executive look
good, because a popular president or governor may be able to help other party members win office.
Even so, partisans in the legislature cannot be expected to simply obey the executive’s orders. First,
legislators may serve a constituency that disagrees with the executive on key matters of policy. If the
issue is important enough to voters, as in the case of gun control or abortion rights, an office holder may
feel his or her job will be in jeopardy if he or she too closely follows the party line, even if that means
disagreeing with the executive. A good example occurred when the Civil Rights Act of 1964, which
desegregated public accommodations and prohibited discrimination in employment on the basis of race,
was introduced in Congress. The bill was supported by Presidents John F. Kennedy and Lyndon Johnson,
both of whom were Democrats. Nevertheless, many Republicans, such as William McCulloch, a
conservative representative from Ohio, voted in its favor while many southern Democrats opposed it.50
A second challenge is that each house of the legislature has its own leadership and committee structure,
and those leaders may not be in total harmony with the president. Key benefits like committee
appointments, leadership positions, and money for important projects in their home district may hinge
on legislators following the lead of the party. These pressures are particularly acute for the majority
party, so named because it controls more than half the seats in one of the two chambers. The Speaker
of the House and the Senate majority leader, the majority party’s congressional leaders, have significant
tools at their disposal to punish party members who defect on a particular vote. Finally, a member of
the minority Get Connected! Chapter 9 | Political Parties 351 party must occasionally work with the
opposition on some issues in order to accomplish any of his or her constituency’s goals. This is especially
the case in the Senate, which is a super-majority institution. Sixty votes (of the 100 possible) are
required to get anything accomplished, because Senate rules allow individual members to block
legislation via holds and filibusters. The only way to block the blocking is to invoke cloture, a procedure
calling for a vote on an issue, which takes 60 votes. 9.4 Divided Government and Partisan Polarization
Learning Objectives By the end of this section, you will be able to: • Discuss the problems and benefits
of divided government • Define party polarization • List the main explanations for partisan polarization
• Explain the implications of partisan polarization In 1950, the American Political Science Association’s
Committee on Political Parties (APSA) published an article offering a criticism of the current party
system. The parties, it argued, were too similar. Distinct, cohesive political parties were critical for any
well-functioning democracy. First, distinct parties offer voters clear policy choices at election time.
Second, cohesive parties could deliver on their agenda, even under conditions of lower bipartisanship.
The party that lost the election was also important to democracy because it served as the “loyal
opposition” that could keep a check on the excesses of the party in power. Finally, the paper suggested
that voters could signal whether they preferred the vision of the current leadership or of the opposition.
This signaling would keep both parties accountable to the people and lead to a more effective
government, better capable of meeting the country’s needs. But, the APSA article continued, U.S.
political parties of the day were lacking in this regard. Rarely did they offer clear and distinct visions of
the country’s future, and, on the rare occasions they did, they were typically unable to enact major
reforms once elected. Indeed, there was so much overlap between the parties when in office that it was
difficult for voters to know whom they should hold accountable for bad results. The article concluded by
advocating a set of reforms that, if implemented, would lead to more distinct parties and better
government. While this description of the major parties as being too similar may have been accurate in
the 1950s; that is no longer the case.51 THE PROBLEM OF DIVIDED GOVERNMENT The problem of
majority versus minority politics is particularly acute under conditions of divided government. Divided
government occurs when one or more houses of the legislature are controlled by the party in opposition
to the executive. Unified government occurs when the same party controls the executive and the
legislature entirely. Divided government can pose considerable difficulties for both the operations of the
party and the government as a whole. It makes fulfilling campaign promises extremely difficult, for
instance, since the cooperation (or at least the agreement) of both Congress and the president is
typically needed to pass legislation. Furthermore, one party can hardly claim credit for success when the
other side has been a credible partner, or when nothing can be accomplished. Party loyalty may be
challenged too, because individual politicians might be forced to oppose their own party agenda if it will
help their personal reelection bids. Divided government can also be a threat to government operations,
although its full impact remains unclear.52 For example, when the divide between the parties is too
great, government may shut down. A 1976 dispute between Republican president Gerald Ford and a
Democrat-controlled Congress over the issue of funding for certain cabinet departments led to a ten-day
shutdown of the government (although the federal government did not cease to function entirely). But
beginning in the 1980s, the interpretation 352 Chapter 9 | Political Parties This OpenStax book is
available for free at http://cnx.org/content/col26739/1.4 that Republican president Ronald Reagan’s
attorney general gave to a nineteenth-century law required a complete shutdown of federal
government operations until a funding issue was resolved (Figure 9.13).53 Clearly, the parties’
willingness to work together and compromise can be a very good thing. However, the past several
decades have brought an increased prevalence of divided government. Since 1969, the U.S. electorate
has sent the president a Congress of his own party in only seven of twenty-three congressional
elections, and during George W. Bush’s first administration, the Republican majority was so narrow that
a combination of resignations and defections gave the Democrats control before the next election could
be held. Over the short term, however, divided government can make for very contentious politics. A
wellfunctioning government usually requires a certain level of responsiveness on the part of both the
executive and the legislative branches. This responsiveness is hard enough if government is unified
under one party. During the presidency of Democrat Jimmy Carter (1977–1980), despite the fact that
both houses of Congress were controlled by Democratic majorities, the government was shut down on
five occasions because of conflict between the executive and legislative branches.54 Shutdowns are
even more likely when the president and at least one house of Congress are of opposite parties. During
the presidency of Ronald Reagan, for example, the federal government shut down eight times; on seven
of those occasions, the shutdown was caused by disagreements between Reagan and the Republicancontrolled Senate on the one hand and the Democrats in the House on the other, over such issues as
spending cuts, abortion rights, and civil rights.55 More such disputes and government shutdowns took
place during the administrations of George H. W. Bush, Bill Clinton, and Barack Obama, when different
parties controlled Congress and the presidency. The most recent government shutdown, the longest in
U.S. history, began in December 2018 under the 115th Congress, when the presidency and both houses
were controlled by Republican majorities, but continued into the 116th, which features a Democratically
controlled House and a Republican Senate. For the first few decades of the current pattern of divided
government, the threat it posed to the government appears to have been muted by a high degree of
bipartisanship, or cooperation through compromise. Many pieces of legislation were passed in the
1960s and 1970s with reasonably high levels of support from both parties. Most members of Congress
had relatively moderate voting records, with regional differences within parties that made
bipartisanship on many issues more likely. Figure 9.13 In the early 1980s, Republican president Ronald
Reagan (left) and Democratic Speaker of the House Tip O’Neil (right) worked together to pass key pieces
of legislation, even though they opposed each other on several issues. (credit: Ronald Reagan
Presidential Library & Museum) For example, until the 1980s, northern and midwestern Republicans
were often fairly progressive, supporting racial equality, workers’ rights, and farm subsidies. Southern
Democrats were frequently quite socially and racially conservative and were strong supporters of states’
rights. Cross-party cooperation on these issues was fairly frequent. But in the past few decades, the
number of moderates in both houses of Congress has declined. This has made it more difficult for party
leadership to work together on a range Chapter 9 | Political Parties 353 of important issues, and for
members of the minority party in Congress to find policy agreement with an opposing party president.
THE IMPLICATIONS OF POLARIZATION The past thirty years have brought a dramatic change in the
relationship between the two parties as fewer conservative Democrats and liberal Republicans have
been elected to office. As political moderates, or individuals with ideologies in the middle of the
ideological spectrum, leave the political parties at all levels, the parties have grown farther apart
ideologically, a result called party polarization. In other words, at least organizationally and in
government, Republicans and Democrats have become increasingly dissimilar from one another (Figure
9.14). In the party-in-government, this means fewer members of Congress have mixed voting records;
instead they vote far more consistently on issues and are far more likely to side with their party
leadership.56 It also means a growing number of moderate voters aren’t participating in party politics.
Either they are becoming independents, or they are participating only in the general election and are
therefore not helping select party candidates in primaries. Figure 9.14 The number of moderates has
dropped since 1973 as both parties have moved toward ideological extremes. What is most interesting
about this shift to increasingly polarized parties is that it does not appear to have happened as a result
of the structural reforms recommended by APSA. Rather, it has happened because moderate politicians
have simply found it harder and harder to win elections. There are many conflicting theories about the
causes of polarization, some of which we discuss below. But whatever its origin, party polarization in the
United States does not appear to have had the net positive effects that the APSA committee was hoping
for. With the exception of providing voters with more distinct choices, positives of polarization are hard
to find. The negative impacts are many. For one thing, rather than reducing interparty conflict,
polarization appears to have only amplified it. For example, the Republican 354 Chapter 9 | Political
Parties This OpenStax book is available for free at http://cnx.org/content/col26739/1.4 Party (or the
GOP, standing for Grand Old Party) has historically been a coalition of two key and overlapping factions:
pro-business rightists and social conservatives. The GOP has held the coalition of these two groups
together by opposing programs designed to redistribute wealth (and advocating small government)
while at the same time arguing for laws preferred by conservative Christians. But it was also willing to
compromise with pro-business Democrats, often at the expense of social issues, if it meant protecting
long-term business interests. Recently, however, a new voice has emerged that has allied itself with the
Republican Party. Born in part from an older third-party movement known as the Libertarian Party, the
Tea Party is more hostile to government and views government intervention in all forms, and especially
taxation and the regulation of business, as a threat to capitalism and democracy. It is less willing to
tolerate interventions in the market place, even when they are designed to protect the markets
themselves. Although an anti-tax faction within the Republican Party has existed for some time, some
factions of the Tea Party movement are also active at the intersection of religious liberty and social
issues, especially in opposing such initiatives as same-sex marriage and abortion rights.57 The Tea Party
has argued that government, both directly and by neglect, is threatening the ability of evangelicals to
observe their moral obligations, including practices some perceive as endorsing social exclusion.
Although the Tea Party is a movement and not a political party, 86 percent of Tea Party members who
voted in 2012 cast their votes for Republicans.58 Some members of the Republican Party are closely
affiliated with the movement, and before the 2012 elections, Tea Party activist Grover Norquist exacted
promises from many Republicans in Congress that they would oppose any bill that sought to raise
taxes.59 The inflexibility of Tea Party members has led to tense floor debates and was ultimately
responsible for the 2014 primary defeat of Republican majority leader Eric Cantor and the 2015
resignation of the sitting Speaker of the House John Boehner. In 2015, Chris Christie, John Kasich, Ben
Carson, Marco Rubio, and Ted Cruz, all of whom were Republican presidential candidates, signed
Norquist’s pledge as well (Figure 9.15). Figure 9.15 Vying for the Republican nomination, 2016
presidential candidates Ted Cruz (a) and John Kasich (b), like many other Republicans, signed a pledge
not to raise taxes if elected. Movements on the left have also arisen. The Occupy Wall Street movement
was born of the government’s Chapter 9 | Political Parties 355 response to the Great Recession of 2008
and its assistance to endangered financial institutions, provided through the Troubled Asset Relief
Program, TARP (Figure 9.16). The Occupy Movement believed the recession was caused by a failure of
the government to properly regulate the banking industry. The Occupiers further maintained that the
government moved swiftly to protect the banking industry from the worst of the recession but largely
failed to protect the average person, thereby worsening the growing economic inequality in the United
States. Figure 9.16 On September 30, 2011, Occupy Wall Street protesters marched to the headquarters
of the New York Police Department to protest police brutality that occurred in response to the
movement’s occupation of Zuccotti Park in Lower Manhattan. (credit: modification of work by David
Shankbone) While the Occupy Movement itself has largely fizzled, the anti-business sentiment to which
it gave voice continues within the Democratic Party, and many Democrats have proclaimed their
support for the movement and its ideals, if not for its tactics.60 Champions of the left wing of the
Democratic Party, however, such as former presidential candidate Senator Bernie Sanders and
Massachusetts senator Elizabeth Warren, have ensured that the Occupy Movement’s calls for more
social spending and higher taxes on the wealthy remain a prominent part of the national debate. Their
popularity, and the growing visibility of race issues in the United States, have helped sustain the left
wing of the Democratic Party. Bernie Sanders’ presidential run made these topics and causes even more
salient, especially among younger voters. This reality led Hillary Clinton to move left during the
primaries and attempt to win people over. However, the left never warmed up to Clinton after Sanders
exited the race. After Clinton lost to Trump, many on the left blamed Clinton for not going far enough
left, and they further claimed that Sanders would have had a better chance at beating Trump.61
Unfortunately, party factions haven’t been the only result of party polarization. By most measures, the
U.S. government in general and Congress in particular have become less effective in recent years.
Congress has passed fewer pieces of legislation, confirmed fewer appointees, and been less effective at
handling the national purse than in recent memory. If we define effectiveness as legislative productivity,
the 106th Congress (1999–2000) passed 463 pieces of substantive legislation (not including
commemorative legislation, such as bills proclaiming an official doughnut of the United States). The
107th Congress (2000–2001) passed 294 such pieces of legislation. By 2013–2014, the total had fallen to
212.62 Perhaps the clearest sign of Congress’ ineffectiveness is that the threat of government shutdown
has become a constant. Shutdowns occur when Congress and the president are unable to authorize and
appropriate funds before the current budget runs out. This is now an annual problem. Relations
between 356 Chapter 9 | Political Parties This OpenStax book is available for free at
http://cnx.org/content/col26739/1.4 the two parties became so bad that financial markets were sent
into turmoil in 2014 when Congress failed to increase the government’s line of credit before a key
deadline, thus threatening a U.S. government default on its loans. While any particular trend can be the
result of multiple factors, the decline of key measures of institutional confidence and trust suggest the
negative impact of polarization. Public approval ratings for Congress have been near single digits for
several years, and a poll taken in February 2016 revealed that only 11 percent of respondents thought
Congress was doing a “good or excellent job.”63 In the wake of the Great Recession, President Obama’s
average approval rating remained low for several years, despite an overall trend in economic growth
since the end of 2008, before he enjoyed an uptick in support during his final year in office.64 Typically,
economic conditions are a significant driver of presidential approval, suggesting the negative effect of
partisanship on presidential approval. THE CAUSES OF POLARIZATION Scholars agree that some degree
of polarization is occurring in the United States, even if some contend it is only at the elite level. But
they are less certain about exactly why, or how, polarization has become such a mainstay of American
politics. Several conflicting theories have been offered. The first and perhaps best argument is that
polarization is a party-in-government phenomenon driven by a decades-long sorting of the voting
public, or a change in party allegiance in response to shifts in party position.65 According to the sorting
thesis, before the 1950s, voters were mostly concerned with state-level party positions rather than
national party concerns. Since parties are bottom-up institutions, this meant local issues dominated
elections; it also meant national-level politicians typically paid more attention to local problems than to
national party politics. But over the past several decades, voters have started identifying more with
national-level party politics, and they began to demand their elected representatives become more
attentive to national party positions. As a result, they have become more likely to pick parties that
consistently represent national ideals, are more consistent in their candidate selection, and are more
willing to elect office-holders likely to follow their party’s national agenda. One example of the way
social change led to party sorting revolves around race. The Democratic Party returned to national
power in the 1930s largely as the result of a coalition among low socio-economic status voters in
northern and midwestern cities. These new Democratic voters were religiously and ethnically more
diverse than the mostly white, mostly Protestant voters who supported Republicans. But the southern
United States (often called the “Solid South”) had been largely dominated by Democratic politicians
since the Civil War. These politicians agreed with other Democrats on most issues, but they were more
evangelical in their religious beliefs and less tolerant on racial matters. The federal nature of the United
States meant that Democrats in other parts of the country were free to seek alliances with minorities in
their states. But in the South, African Americans were still largely disenfranchised well after Franklin
Roosevelt had brought other groups into the Democratic tent. The Democratic alliance worked relatively
well through the 1930s and 1940s when post-Depression politics revolved around supporting farmers
and helping the unemployed. But in the late 1950s and early 1960s, social issues became increasingly
prominent in national politics. Southern Democrats, who had supported giving the federal government
authority for economic redistribution, began to resist calls for those powers to be used to restructure
society. Many of these Democrats broke away from the party only to find a home among Republicans,
who were willing to help promote smaller national government and greater states’ rights.66 This shift
was largely completed with the rise of the evangelical movement in politics, when it shepherded its
supporters away from Jimmy Carter, an evangelical Christian, to Ronald Reagan in the 1980 presidential
election. At the same time social issues were turning the Solid South towards the Republican Party, they
were having the oppo…

Calculate your order
Pages (275 words)
Standard price: $0.00
Client Reviews
4.9
Sitejabber
4.6
Trustpilot
4.8
Our Guarantees
100% Confidentiality
Information about customers is confidential and never disclosed to third parties.
Original Writing
We complete all papers from scratch. You can get a plagiarism report.
Timely Delivery
No missed deadlines – 97% of assignments are completed in time.
Money Back
If you're confident that a writer didn't follow your order details, ask for a refund.

Calculate the price of your order

You will get a personal manager and a discount.
We'll send you the first draft for approval by at
Total price:
$0.00
Power up Your Academic Success with the
Team of Professionals. We’ve Got Your Back.
Power up Your Study Success with Experts We’ve Got Your Back.
WeCreativez WhatsApp Support
Our customer support team is here to answer your questions. Ask us anything!
? Hi, how can I help?